Difference between revisions of "2020 AMC 12B Problems/Problem 23"

(Solution)
m (Solutions)
(22 intermediate revisions by 8 users not shown)
Line 1: Line 1:
 
+
== Problem ==
 
 
==Problem 23==
 
 
 
 
How many integers <math>n \geq 2</math> are there such that whenever <math>z_1, z_2, ..., z_n</math> are complex numbers such that
 
How many integers <math>n \geq 2</math> are there such that whenever <math>z_1, z_2, ..., z_n</math> are complex numbers such that
  
Line 10: Line 7:
 
<math>\textbf{(A)}\ 1 \qquad\textbf{(B)}\ 2 \qquad\textbf{(C)}\ 3 \qquad\textbf{(D)}\ 4 \qquad\textbf{(E)}\ 5</math>
 
<math>\textbf{(A)}\ 1 \qquad\textbf{(B)}\ 2 \qquad\textbf{(C)}\ 3 \qquad\textbf{(D)}\ 4 \qquad\textbf{(E)}\ 5</math>
  
[[2020 AMC 12B Problems/Problem 23|Solution]]
+
== Solution ==
 
 
==Solution==
 
  
 
For <math>n=2</math>, we see that if <math>z_{1}+z_{2}=0</math>, then <math>z_{1}=-z_{2}</math>, so they are evenly spaced along the unit circle.
 
For <math>n=2</math>, we see that if <math>z_{1}+z_{2}=0</math>, then <math>z_{1}=-z_{2}</math>, so they are evenly spaced along the unit circle.
  
For <math>n=3</math>, WLOG, we can set <math>z_{1}=1</math>. Notice that now <math>\Re(z_{2}+z{3})=-1</math> and <math>\Im\{z_{2}\}=-\Im\{z_{3}\}</math>. This forces <math>z_{2}</math> and <math>z_{3}</math> to be equal to <math>e^{i\frac{2\pi}{3}}</math> and <math>e^{-i\frac{2\pi}{3}}</math>, meaning that all three are equally spaced along the unit circle.
+
For <math>n=3</math>, WLOG, we can set <math>z_{1}=1</math>. Notice that now <math>\Re(z_{2}+z_{3})=-1</math> and <math>\Im\{z_{2}\}=-\Im\{z_{3}\}</math>. This forces <math>z_{2}</math> and <math>z_{3}</math> to be equal to <math>e^{i\frac{2\pi}{3}}</math> and <math>e^{-i\frac{2\pi}{3}}</math>, meaning that all three are equally spaced along the unit circle.
  
 
We can now show that we can construct complex numbers when <math>n\geq 4</math> that do not satisfy the conditions in the problem.
 
We can now show that we can construct complex numbers when <math>n\geq 4</math> that do not satisfy the conditions in the problem.
  
Suppose that the problem condition holds for some <math>n=k</math>. We can now add two points <math>z_{k+1}</math> and <math>z_{k+2}</math> anywhere on the unit circle such that <math>z_{k+1}=-z_{k+2}</math>, which will break the condition. Now that we have shown that <math>n=2</math> and <math>n=3</math> works, by this construction, any <math>n\geq 4</math> does not work, making the answer <math>\boxed{\mathbf(B) 2}</math>.
+
Suppose that the condition in the problem holds for some <math>n=k</math>. We can now add two points <math>z_{k+1}</math> and <math>z_{k+2}</math> anywhere on the unit circle such that <math>z_{k+1}=-z_{k+2}</math>, which will break the condition. Now that we have shown that <math>n=2</math> and <math>n=3</math> works, by this construction, any <math>n\geq 4</math> does not work, making the answer <math>\boxed{\textbf{(B)} 2}</math>.
  
 
-Solution by Qqqwerw
 
-Solution by Qqqwerw
 +
 +
== Video Solution ==
 +
On The Spot STEM: https://www.youtube.com/watch?v=JOgSOni5HhM
  
 
==See Also==
 
==See Also==

Revision as of 11:56, 20 June 2022

Problem

How many integers $n \geq 2$ are there such that whenever $z_1, z_2, ..., z_n$ are complex numbers such that

\[|z_1| = |z_2| = ... = |z_n| = 1 \text{    and    } z_1 + z_2 + ... + z_n = 0,\] then the numbers $z_1, z_2, ..., z_n$ are equally spaced on the unit circle in the complex plane?

$\textbf{(A)}\ 1 \qquad\textbf{(B)}\ 2 \qquad\textbf{(C)}\ 3 \qquad\textbf{(D)}\ 4 \qquad\textbf{(E)}\ 5$

Solution

For $n=2$, we see that if $z_{1}+z_{2}=0$, then $z_{1}=-z_{2}$, so they are evenly spaced along the unit circle.

For $n=3$, WLOG, we can set $z_{1}=1$. Notice that now $\Re(z_{2}+z_{3})=-1$ and $\Im\{z_{2}\}=-\Im\{z_{3}\}$. This forces $z_{2}$ and $z_{3}$ to be equal to $e^{i\frac{2\pi}{3}}$ and $e^{-i\frac{2\pi}{3}}$, meaning that all three are equally spaced along the unit circle.

We can now show that we can construct complex numbers when $n\geq 4$ that do not satisfy the conditions in the problem.

Suppose that the condition in the problem holds for some $n=k$. We can now add two points $z_{k+1}$ and $z_{k+2}$ anywhere on the unit circle such that $z_{k+1}=-z_{k+2}$, which will break the condition. Now that we have shown that $n=2$ and $n=3$ works, by this construction, any $n\geq 4$ does not work, making the answer $\boxed{\textbf{(B)} 2}$.

-Solution by Qqqwerw

Video Solution

On The Spot STEM: https://www.youtube.com/watch?v=JOgSOni5HhM

See Also

2020 AMC 12B (ProblemsAnswer KeyResources)
Preceded by
Problem 22
Followed by
Problem 24
1 2 3 4 5 6 7 8 9 10 11 12 13 14 15 16 17 18 19 20 21 22 23 24 25
All AMC 12 Problems and Solutions

The problems on this page are copyrighted by the Mathematical Association of America's American Mathematics Competitions. AMC logo.png